You are on page 1of 20

ANSWERS

Chapter 1
2.45 2.65, 2.65 2.85
7/30
16/30
Approx. .68
Approx. .95
Approx. .815
Approx. 0
y = 9.79; s = 4.14
k = 1: (5.65, 13.93); k = 2: (1.51,
18.07); k = 3: (2.63, 22.21)
1.15 a y = 4.39; s = 1.87
b k = 1: (2.52, 6.26); k = 2: (0.65,
8.13); k = 3: (1.22, 10)
1.17 For Ex. 1.2, range/4 = 7.35; s = 4.14;
for Ex. 1.3, range/4 = 3.04; s = 3.17;
for Ex. 1.4, range/4 = 2.32, s = 1.87.
1.19 y s = 19 < 0

1.5 a
b
c
1.9 a
b
c
d
1.13 a
b

1.21 .84
1.23 a 16%
b Approx. 95%
1.25 a 177
c y = 210.8; s = 162.17
d k = 1: (48.6, 373); k = 2:
1.27
1.29
1.31
1.33
1.35

(113.5, 535.1); k = 3: (275.7,


697.3)
68% or 231 scores; 95% or 323 scores
.05
.025
(0.5, 10.5)
a (172 108)/4 = 16
b y = 136.1; s = 17.1
c a = 136.1 2(17.1) = 101.9;
b = 136.1 + 2(17.1) = 170.3

Chapter 2
2.7 A = {two males} = {(M1 , M2 ),

(M1 ,M3 ), (M2 ,M3 )}


B = {at least one female} = {(M1 ,W1 ),
(M2 ,W1 ), (M3 ,W1 ), (M1 ,W2 ), (M2 ,W2 ),
(M3 ,W2 ), (W1 ,W2 )}
B = {no females} = A; A B = S;
A B = null; A B = A
2.9 S = {A+ , B+ , AB+ , O+ , A , B ,
AB , O }
2.11 a P(E 5 ) = .10; P(E 4 ) = .20
b p = .2
2.13 a E1 = very likely (VL); E2 =
somewhat likely (SL); E3 =
unlikely (U); E4 = other (O)
b No; P(VL) = .24, P(SL) = .24,
P(U) = .40, P(O) = .12
c .48

2.15 a
b
2.17 a
b
c
d
2.19 a
b
c
2.27 a
b
c

.09
.19
.08
.16
.14
.84
(V1 , V1 ), (V1 , V2 ), (V1 , V3 ),
(V2 , V1 ), (V2 , V2 ), (V2 , V3 ),
(V3 , V1 ), (V3 , V2 ), (V3 , V3 )
If equally likely, all have
probability of 1/9.
P(A) = 1/3; P(B) = 5/9;
P(A B) = 7/9;
P(A B) = 1/9
S = {CC, CR, CL, RC, RR, RL,
LC, LR, LL}
5/9
5/9
877

878

Answers

2.29 c 1/15
2.31 a 3/5; 1/15
b 14/15; 2/5
2.33 c 11/16; 3/8; 1/4
2.35 42
2.37 a 6! = 720
b .5
2.39 a 36
b 1/6
2.41 9(10)6
2.43 504 ways
2.45 408,408
2.49 a 8385
b 18,252
c 8515 required
d Yes
2.51 a 4/19,600
b 276/19,600
c 4140/19,600
d 15180/19,600
2.53 a 60 sample points
b 36/60
  = .6

90
10
   + 
90
70
20
= .111
b
10
6
4
(4 12)/1326 = .0362
a .000394
b .00355
364n
a
365n
b .5005
1/56
5/162
a P(A) = .0605
b .001344
c .00029
a 1/3
b 1/5
c 5/7
d 1
e 1/7
a 3/4
b 3/4
c 2/3
a .40
b .37
c .10
d .67
e .6
f .33
g .90
h .27
i .25
.364
a .1
b .9

2.55 a

2.57
2.59
2.61
2.63
2.65
2.67
2.71

2.73
2.77
2.93
2.95

c .6
d 2/3
2.97 a .999
b .9009
2.101 .05
2.103 a .001
b .000125
2.105 .90
2.109 P(A) .9833
2.111 .149
2.113 (.98)3 (.02)
2.115 (.75)4
2.117 a 4(.5)4 = .25
b (.5)4 = 1/16
2.119 a 1/4
b 1/3
2.121 a 1/n

1 1
;
n n
3
c
7
1/12
a .857
c No; .8696
d Yes
.4
.9412
a .57
b .18
c .3158
d .90
a 2/5
b 3/20
P(Y = 0) = (.02)3 ;
P(Y = 1) = 3(.02)2 (.98);
P(Y = 2) = 3(.02)(.98)2 ;
P(Y = 3) = (.98)3
P(Y = 2) = 1/15; P(Y = 3) = 2/15;
P(Y = 4) = 3/15; P(Y = 5) = 4/15;
P(Y = 6) = 5/15
18!
.0083
a .4
b .6
c .25
4[ p 4 (1 p) + p(1 p)4 ]
.313
a .5
b .15
c .10
d .875

2.125
2.127
2.129
2.133
2.135

2.137
2.139

2.141
2.145
2.147
2.149
2.151
2.153
2.155

Answers 879

2.157 .021
2.161 P(R 3) = 12/66
2.163 P(A) = 0.9801
P(B) = .9639

2.165 .916
2.167 P(Y = 1) = 35/70 = .5;

P(Y = 2) = 20/70 = 2/7;


P(Y = 3) = 10/70;
P(Y = 4) = 4/70; P(Y = 5) = 1/70
2.169 a (4!)3 = 13,824

b 3456/13,824 = .25
2.173 .25
2.177 a .364
b .636
n
c (49/50)
.60, so n is at most 25
 
1 6
= .3125
 2 10
1
b 27
2

2.179 a 20

Chapter 3
P(Y = 2) = .3894,
P(Y = 3) = .1406
c P(Y = 1) = .3594
d = E(Y ) = 1.56, 2 = .7488,
= 0.8653
e (.1706, 3.2906),
P(0 Y 3) = 1

3.1 P(Y = 0) = .2, P(Y = 1) = .7,


3.3
3.5
3.7

3.9

3.11

3.13

3.15

P(Y = 2) = .1
2
1
1
p(2) = , p(3) = , p(4) =
6
6
2
3
1
2
p(0) = , p(1) = , p(3) =
6
6
6
6
3
3!
=
, p(2) =
,
p(0) =
27
27
27
3
18
6

=
p(1) = 1
27 27
27
a P(Y = 3) = .000125,
P(Y = 2) = .007125,
P(Y = 1) = .135375,
P(Y = 0) = .857375
c P(Y > 1) = .00725
12
8
, P(X = 1) =
,
P(X = 0) =
27
27
1
6
, P(X = 3) =
,
P(X = 2) =
27
27
2744
,
P(Y = 0) =
3375
588
,
P(Y = 1) =
3375
14
,
P(Y = 2) =
3375
1
, Z = X + Y,
P(Y = 3) =
3375
54
27
, P(Z = 1) =
,
P(Z = 0) =
125
125
8
36
, P(Z = 3) =
P(Z = 2) =
125
125
27
1
7
2
,
E(Y ) = , E(Y ) = , V (Y ) =
4
4
16
1
cost =
4
a P(Y = 0) = .1106,
P(Y = 1) = .3594,

3.17 = E(Y ) = .889,

2 = V (Y ) = E(Y 2 )[E(Y )]2 = .321,


= 0.567, ( 2 ,
+ 2 ) = (.245, 2.023),
P(0 Y 2) = 1

3.19
3.21
3.23
3.25

C = $85
13,800.388
$.31
Firm I : E (prot) = $60,000
E(total prot) = $120,000

3.27 $510
3.35 .4; .3999
3.39 a .1536;
b .9728
3.41 .000
3.43 a .1681
b .5282
3.45 P(alarm functions) = 0.992
3.49 a .151
b .302
3.51 a .51775
b .4914
3.53 a .0156
b .4219
c 25%

880

Answers

3.57 $185,000
3.59 $840
3.61 a .672
b .672
c 8
3.67 .07203
3.69 Y is geometric with p = .59
3.73 a .009
b .01
3.75 a .081
b .81
3.81 2
3.83
3.87
3.91
3.93
3.95
3.97

3.99

9
1
, p(3) =
30
30
3.113 P(Y 1) = .187
3
1
1
3.115 p(0) = , p(1) = , p(2) =
5
5
5
3.117 a P(Y = 0) = .553
b E(T ) = 9.5, V (T ) = 28.755,
= 5.362
p(2) =

3.119 .016
3.121 a .090
b .143
c .857
d .241


3.123 .1839
1 n1 5
3.125 E(S) = 7, V (S) = 700; no
n
n
 
p ln( p)
1
3.127 .6288
=
E
Y
1 p
3.129 23 seconds
$150; 4500
3.131 .5578
a .04374
3.133 .1745
b .99144
3.135 .9524
.1
3.137 .1512
a .128
3.139 40
b .049
2
3.141 $1300
c = 15, = 60
y!
3.149 Binomial, n = 3 and p = .6
pr q y+1r ,
p(x) =
(r 1)!(y r + 1)!
3.151 Binomial, n = 10 and p = .7,

y = r 1, r , r + 1, . . .
5
3.101 a
11
r
b
y0
1
3.103
42
3.105 b .7143
c = 1.875,
= .7087

3.107 hypergeometric with N = 6, n = 2,


and r = 4.

3.109 a .0238
b .9762
c .9762
14
14
, p(1) =
,
30
30
2
p(2) =
30
5
15
b p(0) = , p(1) = ,
30
30

3.111 a p(0) =

P(Y 5) = .1503

3.153 a Binomial, n = 5 and p = .1


1

b Geometric, p =
2
c Poisson, = 2

7
3
5
V (Y ) =
9
2
3
1
p(1) = , p(2) = , p(3) =
6
6
6
.64
C = 10

3.155 a E(Y ) =
b
c

3.167 a
b
3.169 d p(1) = 1/(2k 2 ),

p(0) = 1 (1/k 2 ), p(1) = 1(2k 2 )

3.171 (85, 115)


3
3
1
3.173 a p(0) = , p(1) = , p(2) = ,
8
8
1
p(3) =
8
c E(Y ) = 1.5, V (Y ) = .75,
= .866

Answers 881

3.175 a 38.4
b 5.11
3.177 (61.03, 98.97)
3.179 No, P(Y 350)

1
= .1126.
(2.98)2

3.181
a
b
c
d
e

p = Fraction defective P(acceptance)


0
1
.10
.5905
.30
.1681
.50
.0312
1.0
0

3.185 a .2277
b Not unlikely
3.187 a .023
b 1.2
c $1.25
3.189 1 (.99999)10,000
3.191 V (Y ) = .4
3.193 .476

3.195 a .982
b P(W 1) = 1 e12
3.197 a .9997
b n=2
3.199 a .300
b .037
3.201 (18.35, 181.65)
3.203 a E[Y (t)] = k(e2t et )
b 3.2974, 2.139
3.205 .00722
3.207 a p(2) = .084, P(Y 2) = .125
b P(Y > 10) = .014
3.209 .0837
3.211 3
3.213 a .1192
b .117
3.215 a n[1 + k(1 .95k )]
b g(k) is minimized at k = 5 and
g(5) = .4262.

c .5738N

Chapter 4
4.7 a P(2 Y < 5) = 0.591,
b
c
4.9 a
b

P(2 < Y < 5) = .289, so


not equal
P(2 Y 5) = 0.618,
P(2 < Y 5) = 0.316, so
not equal
Y is not a continuous random
variable, so the earlier results
do not hold.
Y is a discrete random variable
These values are 2, 2.5, 4, 5.5, 6,
and 7.

1
1
,
8
16
1
5
, p(5.5) = ,
p(4) =
16
8
5
1
, p(7) =
p(6) =
16
16
d .5 = 4
1
4.11 a c =
2
y2
b F(y) = , 0 y 2
4

c p(2) = , p(2.5) =

d .75
e .75

4.13 a

b
c
4.15 a

0
y<0

0y1
2
F(y) =

1 < y 1.5
y

1
y > 1.5
.125
.575
For b 0, f (y) 0; also,
#
f (y) = 1

b
y

b F(y) = 1 , for y b;
c
d
4.17 a
b

0 elsewhere.
b
(b + c)
(b + c)
(b + d)
3
c=
2
y2
y3
+ , for 0 y 1
F(y) =
2
2

882

Answers

d F(1) = 0, F(0) = 0, F(1) = 1


3
e
16
104
f
123

4.19

4.21
4.25
4.27
4.29
4.31
4.33

4.37
4.39
4.45

4.47

4.49
4.51
4.53

4.55
4.57
4.59

0
y0

.125
0<y<2
a f (y) =
.125y 2 y < 4

0
y4
7
b
16
13
c
16
7
d
9
E(Y ) = .708, V (Y ) = .0487
E(Y ) = 31/12, V (Y ) = 1.160
$4.65, .012
1
E(Y ) = 60, V (Y ) =
3
E(Y ) = 4
a E(Y ) = 5.5, V (Y ) = .15
b Using Tchebysheffs theorem,
the interval is (5, 6.275).
c Yes; P(Y ) = .5781
E(Y ) = 0
.5; .25
2
a P(Y < 22) = = .4
5
1
b P(Y > 24) = = .2
5
3
a P(Y > 2) =
4

4
+9
b c0 + c1
3
3
4
1
3
1
a
8
1
b
8
1
c
4
2
a
7
b  = .015,
V (Y ) = .00041
3
D = .0000065 ,
E
6 

V
D 3 = .0003525 2
6
a z0 = 0

b z 0 = 1.10
c z 0 = 1.645
d z 0 = 2.576
4.63 a P(Z > 1) = .1587
b The same answer is obtained.
4.65 $425.60
4.67 = 3.000 in.
4.69 .2660
4.71 a .9544
b .8297
4.73 a .406
b 960.5 mm
4.75 = 7.301
4.77 a 0.758
b 22.2
4.87 a .05 = .70369.
b .05 = .35185
4.89 a = .8
b P(Y 1.7) = .8806
4.91 a .1353
b 460.52 cfs
4.93 a .5057
b 1936
4.97 .3679
4.99 a .7358
4.101 a E(Y ) = 1.92
b P(Y > 3) = .21036
d P(2 Y 3) = .12943
4.103 E(A) = 200, V (A) = 200,000 2
4.105 a E(Y ) = 3.2, V (Y ) = 6.4
b P(Y > 4) = .28955
4.107 a (0, 9.657), because Y must
be positive.

b P(Y < 9.657) = .95338

4.109 E(L) = 276,


 = 47,664
 V(L)

1
/ () if > 0
 +
2
( 12 )
1
if > 1,
e
( 1)
()
1
1
if > , 2
2 ( 1)( 2)
if > 2
a k = 60
b .95 = 0.84684
1
3
E(Y ) = , V (Y ) =
5
25
52
, V (C) = 29.96
E(C) =
3
a .75
b .2357
a c = 105

4.111 d

4.123
4.125
4.129
4.131
4.133

Answers 883

b =
8
c = .1614
d .02972

4.139 m X (t) = exp{t (43)+(1/2)(9 2 t 2 )}

4.141
4.143
4.145

4.147
4.149
4.151

4.153
4.155
4.157

4.159

normal, E(X ) = 4 3, V (X ) = 9 2 ,
uniqueness of moment-generating
functions t
e 2 et1
m(t) =
t (2 1 )
, 2
2
a
5
1
b
(t + 1)
c 1
1
=
2
1
The value 2000 is only .53 standard
deviation above the mean. Thus, we
would expect C to exceed 2000
fairly often.
(6.38, 28.28)
$113.33
a
F(x) =
x <0
0,
(1/100)ex/100 , 0 x < 200

1,
x 200
b 86.47
a
F1 (y) =
0
y<0

.1
= .4 0 y < 5 ;

.1 + .15
1
y .5
F2 (y) =

4.161
4.163
4.165

4.167
4.169
4.171
4.173
4.175
4.179
4.181
4.183

4.187
4.189
4.191
4.193
4.195

0
y<0

2
4y /3
0 y < .5
(4y 1)/3 .5 y < 1

1
y1
b F(y) = 0.25F1 (y) + 0.75F2 (y)
c E(Y ) = .533, V (Y ) = .076
.9 = 85.36
1 (.927)5 = .3155
a c=4
b E(Y ) = 1, V (Y ) = .5
1
c m(t) =
,t <2
(1 .5t)2
( + )(k + )
E(Y k ) =
()(k + + )
e2.5 = .082
1
1
a E(W ) = , V (W ) =
2
4
b 1 e6
2
f (r ) = 2r er , r > 0

2 = 1.414
k = (.4)1/3 = .7368
m(t) = exp(t 2 /2); 0; 1
a E(Y ) = 598.74 g
V (Y ) = e22 (e16 1)104
b (0, 3,570,236.1)
c .8020
a e2.5 = .082
b .0186
E(Y ) = 0. Also, it is clear that
1
V (Y ) = E(Y 2 ) =
.
n1
4
c 1e
150
a 12
b w = 120

Chapter 5
0

y1
1

1
9

2
9

1
9

2
9

2
9

1
9

5.1 a

y2

b F(1, 0) =

5.3

1
3

4 3
2

y1
y2 3 y1 y2

, where
9

3

0 y1 , 0 y2 , and y1 + y2 3.
.1065
.5
.00426
.8009
k=6
31
b
64
29
5.11 a
32
1
b
4

5.5 a
b
5.7 a
b
5.9 a

884

Answers

1 1
,
=
2 2


1
, 2 =
F
2
.65625
e1 2e2
1
2
e1


5.13 a F
b
c
5.15 a
b
c
5.17 .50
5.19 a

y1

p1 (y1 )

b
c
2
4
9

1
9

b No
5.21 a Hypergeometric with N = 9,
b
c
5.23 a
b
c
5.25 a
b
c
d
e
f
g
5.27 a
b
c
d
e
5.29 a

b
5.31 a

n = 3, and r = 4.
2
3
8
15
3 3
f 2 (y2 ) = y22 , 0 y2 1
2 2
Dened over y2 y1 1 if y2 0
1
3
f 1 (y1 ) = ey1 , y1 > 0;
f 2 (y2 ) = ey2 , y2 > 0
P(1 < Y1 < 2.5) = P(1 < Y2 <
2.5) = e1 e2.5 = .2858
y2 > 0
f (y1 |y2 ) = f 1 (y1 ) = ey1 , y1 > 0
f (y2 |y1 ) = f 2 (y2 ) = ey2 , y2 > 0
same
same
f 1 (y1 ) = 3(1 y1 )2 , 0 y1 1;
f 2 (y2 ) = 6y2 (1 y2 ), 0 y2 1
32
63
1
f (y1 |y2 ) = , 0 y1 y2 ,
y2
if y2 1
2(1 y2 )
f (y2 |y1 ) =
,
(1 y1 )2
y1 y2 1 if y1 0
1
4
f 2 (y2 ) = 2(1 y2 ), 0 y2 1;
f 1 (y1 ) = 1 |y1 |, for
1 y1 1
1
3
f 1 (y1 ) = 20y1 (1 y1 )2 , 0
y1 1

15(1 + y2 )2 y22 ,

1 y2 < 0

15(1 y2 )2 y22 ,
(y2 |y1 ) = 32 y22 (1

0 y2 1

y1 )3 ,
for y1 1 y2 1 y1
.5
f 1 (y1 ) = y1 ey1 , y1 0;
f 2 (y2 ) = ey2 , y2 0
f (y1 |y2 ) = e(y1 y2 ) , y1 y2
f (y2 |y1 ) = 1/y1 , 0 y2 y1

c f
d
5.33 a

1
4
9

b f 2 (y2 ) =

9
16
13
16

5.35 .5
5.37 e1
5.41
5.45
5.47
5.51

5.53
5.55
5.57
5.59
5.61
5.63
5.65
5.69

1
4
No
Dependent
a f (y1 , y2 ) = f 1 (y1 ) f 2 (y2 ) so that
Y1 and Y2 are independent.
b Yes, the conditional probabilities
are the same as the marginal
probabilities.
No, they are dependent.
No, they are dependent.
No, they are dependent.
No, they are dependent.
Yes, they are independent.
1
4
Exponential, mean
 1
1 (y1 +y2 )/3
e
a f (y1 , y2 ) =
,
9
y1 > 0, y2 > 0
b P(Y1 + Y2 1) =
4
1 e1/3 = .0446
3
1

5.71 a

4
23
b
144
4
5.73
3
5.75 a 2
b .0249
c .0249
d 2
e They are equal.
1 1
5.77 a ;
4 2
3
b E(Y12 ) = 1/10, V (Y1 ) = ,
80
3
1
, V (Y2 ) =
E(Y22 ) =
10
20
5
c
4

Answers 885

5.79
5.81
5.83
5.85

5.87
5.89
5.91
5.93

5.95

5.97

5.99
5.101
5.103
5.105
5.107
5.109
5.113

0
1
1

5.115 b V (Y ) = 38.99

c The interval is 14.7 2 38.99 or

a E(Y1 ) = E(Y2 ) = 1 (both

5.117 p1 p2 ,

(0, 27.188)

N n
[ p 1 + p 2 ( p 1 p 2 )2 ]
n(N 1)
5.119 a .0823
n
2n
b E(Y1 ) = , V (Y1 ) =
3
9
n
c Cov(Y2 , Y3 ) =
9
2n
d E(Y2 Y3 ) = 0, V (Y2 Y3 ) =
3
5.121 a .0972
b .2; .072
5.123 .08953
5.125 a .046
b .2262
5.127 a .2759
b .8031
y
5.133 a 2
2
1
b
4
3
5.135 a
2
b 1.25
3
5.137
8
y1
1 0 1 y2
0 1
5.139 a n
1 1 1
2 1
p1 (y1 )
p2 (y2 )
b
3 3 3
3 3

22
5.141 E(Y2 ) = , V (Y2 ) =
Cov(Y1, Y2 ) = 0
2
3
a 2
5.143 m U (t) = (1 t 2 )1/2 , E(U ) = 0,
b Impossible
V (U ) = 1
1
c 4 (a perfect positive linear
5.145
association)
3
11
d 4 (a perfect negative linear
5.147
36
association)
5.149 a f (y1 ) = 3y12 , 0 y1 1
0
3

f (y2 ) = (1 y22 ), 0 y2 1
a
2
4
E(3Y1 + 4Y2 6Y3 ) = 22,
23
b
V (3Y1 + 4Y2 6Y3 ) = 480
44
1
2y1
c f (y1 |y2 ) =
, y2 y1 1
9
(1 y22 )
E(Y1 + Y2 ) = 2/3 and
5
d
1
12
V (Y1 + Y2 ) =
18
5.157 
p(y) =

y 

(11.48, 52.68)

1
y+1
,
E(G) = 42, V (G) = 25; the value $70
y
+1
+1
70 42
y
=
0,
1,
2,
.
.
.
is
= 7.2 standard deviations
5
5.161 E(Y X ) = 1 2 , V (Y X ) =
above the mean, an unlikely value.
12 /n + 22 /m
marginal distributions are
exponential with mean 1)
b V (Y1 ) = V (Y2 ) = 1
c E(Y1 Y2 ) = 0

d E(Y1 Y2 ) = 1 , so
4

Cov(Y1 , Y2 ) =
14
 1


2
,
2
2
+
2
+
e
2
2
a E(Y1 + Y2 ) = 1 + 2
b V (Y1 + Y2 ) = 21 + 22
2
Cov(Y1, Y2 ) = . As the value of Y1
9
increases, the value of Y2 tends to
decrease.
Cov(Y1, Y2 ) = 0
a 0
b Dependent
c 0
d Not necessarily independent
The marginal distributions for Y1
and Y2 are

886

Answers

5.163 b F(y1 , y2 ) =

y1 y2 [1 (1 y1 )(1 y2 )]
c f (y1 , y2 ) =
1 [(1 2y1 )(1 2y2 )],
0 y1 1, 0 y2 1

d Choose two different values for


with 1 1.

5.165 a ( p1 et1 + p2 et2 + p3 et3 )n


b m(t, 0, 0)
c Cov(X 1, X 2 ) = np1 p2

Chapter 6
1u
, 1 u 1
2
u+1
, 1 u 1
b
2
1
c 1, 0 u 1
u
d E(U1 ) = 1/3, E(U2 ) =
1/3, E(U3 ) = 1/6
e E(2Y 1) = 1/3, E(1 2Y ) =
1/3, E(Y 2 ) = 1/6
b $fU (u) =
(u + 4)/100, 4 u 6
1/10,
6 < u 11
c 5.5833 

1 u 3 1/2
fU (u) =
,
16
2
5 u 53
1
a fU (u) = u 1/2 eu/2 ,
2
u0
b U has a gamma distribution with
= 1/2 and = 2 (recall that

(1/2) = ).
a fU (u) = 2u, 0 u 1
b E(U ) = 2/3
c E(Y1 + Y2 ) = 2/3
a fU (u) = 4ue2u , u 0, a gamma
density with = 2
and = 1/2
b E(U ) = 1, V (U ) = 1/2
u
fU (u) = FU (u) = 2 eu/ , u > 0

[ln(1 U )]1/2
y 1
a f (y) =
,0 y

1/
b Y = U

c y = 4 u. The values are 2.0785,


3.229, 1.5036, 1.5610, 2.403.
fU (u) = 4ue2u for u 0
2
2
a f Y (y) = wew / , w 0, which

is Weibull density
 with
 m = 2.
k
b E(Y k/2 ) = 
+ 1 k/2
2

6.1 a

6.3

6.5
6.7

6.9
6.11

6.13
6.15
6.17

6.25
6.27

6.29 a f W (w) =

1
w 1/2 ew/kT w > 0
(kT
)3/2
2
3
b E(W ) = kT
2
2
fU (u) =
,u 0
(1 + u)3
fU (u) = 4(80 31u + 3u 2 ),
4.5 u 5
fU (u) = ln(u), 0 u 1
a m Y1 (t) = 1 p + pet
b m W (t) = E(et W ) = [1 p + pet ]n
fU (u) = 4ue2u , u 0
a Y has a normal distribution
with mean and variance 2 /n
b P(|Y | 1) = .7888
c The probabilities are .8664, .9544,
.9756. So, as the sample size
increases, so does the probability
that P(|Y | 1)
c = $190.27
P(U > 16.0128) = .025
The distribution of Y1 + (n 2 Y2 ) is
binomial with n 1 + n 2 trials and success
probability p = .2
a Binomial (nm, p) where
ni = m
b Binomial (n 1 = n 2 + n n , p)
c Hypergeometric (r = n,
N = n1 + n2 + nn )
P(Y 20) = .077
a f (u 1 , u 2 ) =
1 [u 2 +(u 2 u 1 )2 ]/2
e 1
=
2
1 (2u 2 2u 1 u 2 +u 2 )/2
1
2
e
2
b E(U1 ) = E(Z 1 ) = 0,
E(U2 ) = E(Z 1 + Z 2 ) = 0,
V (U1 ) = V (Z 1 ) = 1,
V (U2 ) = V (Z 1 + Z 2 ) =
V (Z 1 ) + V (Z 2 ) = 2,
Cov(U1 , U2 ) = E(Z 12 ) = 1


6.31
6.33
6.35
6.37
6.39
6.43

6.45
6.47
6.51
6.53

6.55
6.65

3

Answers 887

c Not independent since

6.89 f R (r ) = n(n 1)r n2 (1 r ),

d This is the bivariate normal

6.93 f (w) =


= 0.

6.69

6.73
6.75
6.77

6.81
6.83
6.85
6.87

distribution with 1 = 2 = 0,
1
12 = 1, 22 = 2, and =
2
1
a f (y1 , y2 ) = 2 2 , y1 > 1,
y1 y2
y2 > 1
e No
a g(2) (u) = 2u, 0 u 1
b E(U2 ) = 2/3, V (U2 ) = 1/18
(10/15)5
n!
a
( j 1)!(k 1 j)!(n k)!
j1
y j [yk y j ]k1 j [ yk ]nk
n
0 y j < yk
(n k + 1) j 2

b
(n + 1)2 (n + 2)
(n k + j + 1)(k j) 2

c
(n + 1)2 (n + 2)
9
b 1e
1 (.5)n
.5
a g(1) (y) = e(y4) , y 4

6.95

6.97

6.101
6.103
6.105
6.107
6.109

b E(Y(1) ) = 5

0r 1 

2
1
w ,0w 1
3
w

0u1
2
a fU1 (u) =
1

u>1
2u 2
u
b fU2 (u) = ue , 0 u
c Same as Ex. 6.35.
p(W = 0) = p(0) = .0512,
p(1) = .2048, p(2) = .3264,
p(3) = .2656, p(4) = .1186,
p(5) = .0294, p(6) = .0038,
p(7) = .0002
fU (u) = 1, 0 u 1 Therefore, U has
a uniform distribution on (0, 1)
1
, < u1 <
(1 + u 21 )
1
u 1 (1 u)1 , 0 < u < 1
B(, )
1

0u<1

4 u
fU (u) =
1


1u9
8 u
P(U = C1 C3 ) = .4156;
P(U = C2 C3 ) = .5844

Chapter 7
7.9 a .7698
b For n = 25, 36, 69, and 64, the

7.11
7.13
7.15

7.17
7.19
7.21
7.27

probabilities are (respectively)


.8664, .9284, .9642, .9836.
c The probabilities increase with n.
d Yes
.8664
.9876
a E( X Y ) = 1 2
b V ( X Y ) = 12 /m + 22 /n
c The two sample sizes should be at
least

6 18.2
P
Z

6
= .57681
i=1 i
P(S 2 .065) = .10
a b = 2.42
b a = .656
c .95
a .17271
b .23041
d .40312

7.31 a
c
d
7.35 a
b
c
7.39

7.43
7.45
7.47
7.49
7.51
7.53
7.55

5.99, 4.89, 4.02, 3.65, 3.48, 3.32


13.2767
13.2767/3.32 4
E(F) = 1.029
V (F) = .076
3 is 7.15 standard deviations above
this mean; unlikely value.
a normal, E( ) = =
c1 1 + c
2 2 + + ck k

c2
c2
c2
V ( ) = 1 + 2 + + k 2
n1
n2
nk
b 2 with n 1 + n 2 + + n k k df
c t with n 1 + n 2 + + n k k df
.9544
.0548
153
.0217
664
b Y is approximately normal: .0132.
a random sample; approximately 1.
b .1271

888

Answers

7.57
7.59
7.61
7.63
7.65
7.67

7.71
7.73
7.75
7.77
7.79

7.81

.0062
.0062
n = 51
56 customers
a Exact: .91854; normal
approximation: .86396.
a n = 5 (exact: .99968;
approximate: .95319); n = 10
(exact: .99363; approximate:
.97312); n = 15 (exact: .98194;
approximate: .97613); n = 20
(exact: .96786; approximate:
.96886)
a n>9
b n > 14, n > 14, n > 36, n > 36,
n > 891, n > 8991
.8980
.7698
61 customers
a Using the normal approximation:
.7486.
b Using the exact binomial
probability: .729.
a .5948

b With p = .2 and .3, the


7.83
7.85
7.87
7.89
7.91

probabilities are .0559 and .0017


respectively.
a .36897
b .48679
.8414
.0041
= 10.15
Since X , Y , and W are normally
.
distributed, so are X , Y , and W
U = E(U ) = .41 +.22 +.43
 2
1
U2 = V (U ) = .16
n1
 2
 2
3
2
+ .16
+ .04
n2
n3

7.95 a F with num. df = 1, denom. df = 9


b F with num. df = 9, denom. df = 1
c c = 49.04
7.97 b .1587
7.101 .8413
7.103 .1587
7.105 .264

Chapter 8
8.3 a B( ) = a + b = (a 1) + b
b Let = ( b)/a
8.5 a MSE( ) = V ( ) = V ( )/a 2
22 c
12 + 22 2c
Y 1
3 92 + 54
2
b [n
1)](Y /n)[1 (Y /n)]
 /(n 
1

a
3n 1
2
=
2
b MSE()
(3n 1)(3n 2)
a (1 2 p)/(n + 2)
np(1 p) + (1 2 p)2
b
(n + 2)2
c p will be close to .5.
MSE( ) = 2
11.5 .99
a 11.3 1.54
b 1.3 1.7
c .17 .08
a .7
b .404
a .601 .031
a .06 .045

8.7 a =
8.9
8.11
8.13
8.15

8.17

8.19
8.21
8.23
8.25
8.27
8.29

8.31 a .03 .041


8.33 .7 .205
8.35 a 20 1.265
b 3 1.855, yes
8.37 1020
645.1


8.39
8.41
8.43
8.45
8.47
8.49
8.57
8.59
8.61
8.63
8.65
8.67
8.69
8.71

2Y
2Y
,
9.48773 .71072
a (Y 2 /5.02389, Y 2 /.0009821)
b Y 2 /.0039321
c Y 2 /3.84146
b [Y(n) ](.95)1/n
a Y /.05132
b 80%
c (2.557, 11.864)
c (3.108, 6.785)
.51 .04
a .78 .021
(15.46, 36.94)
a .78 .026 or (.754, .806)
a .06 .117 or (.057, .177)
a 7.2 .751
b 2.5 .738
.22 .34 or (.12, .56)
n = 100

Answers 889

8.73
8.75
8.77
8.79
8.81
8.83
8.85
8.87
8.91
8.93

8.95
8.99

8.101

n = 2847
n = 136
n = 497
a n = 2998
b n = 1618
60.8 5.701
a 3.4 3.7
b .7 3.32
1 4.72
(.624, .122)
(84.39, 28.93) 1
3
4
+
a 2 X + Y 1.96
n
m
1
3
4
+ , where
b 2 X + Y t/2 S
n 
m

(Yi Y )2 + 1/3 (X i X )2
S2 =
n+m2
(.227,
0 2.196)
(n 1)S 2
a
2
1
0
(n 1)S 2
b
2
2
s = .0286; (.013 .125)

8.103
8.105
8.107
8.109
8.111
8.113
8.115
8.117
8.119
8.121
8.123
8.125

8.129
8.131
8.133

(1.407, 31.264); no
1 2(.0207) = .9586
765 seeds
a .0625 .0237
b 563
n = 38,416
n = 768
(29.30, 391.15)
11.3 1.44
3 3.63
.75 .77
.832 .015
2
S2
a 12 22
S
1
2

S22
S22
b
,
F
v
,v
,/2
S12 Fv2 ,v1 ,/2 S12 1 2
v i = n i 1, i = 1, 2
2(n 1) 4
a
n2
1
c=
n+1
2 4
b
n1 + n2 2

Chapter 9
9.1 1/3; 2/3; 3/5
9.3
9.5
9.7
9.9
9.23
9.25
9.31
9.35

9.47

12n 2
b
(n + 2)(n + 1)2
n1
1/n
a X6 = 1
c need Var(X 2i X 2i1 ) <
b .6826
c No

a Y n is unbiased for .
1 n
b V (Y n ) = 2
2
i=1 i
n
n

ln(Yi ); no

9.75

9.77
9.81
9.83
9.85

i=1

9.57 Yes


1
9.59 3 Y 2 + Y 1



n

n+1
Y(n)
n
3n + 1
Y(n)
9.63 b
3n

2Y 1
, no, not MVUE
9.69 =
1 Y


9.61

1 n
Y 2.
i=1 i
n
1 n
With m 2 =
Y 2 , the MOM
i=1 i
n
1 2m 2
.
estimator of is =
4m 2 1
2
Y
3
2

Y

1
a = Y(n) 1
2
 2
b Y(n) /12
1
a = Y

b E( ) = , V ( ) = 2 /(n)
n
d
Y
n
n i
 i=1

Yi 2 i=1
Yi
2 i=1
e
,
31.4104
10.8508
p A = .30, p B = .38
p C = .32; .08 .1641
Y(n) /2
a Y(1)
c [(/2)1/2n Y(1) , (1 (/2))1/2n Y(1) ]
a 1/Y
b 1/Y

9.71 2 = m 2 =

9.87
9.91
9.93
9.97

890

Answers
1

p (1 p )
n1
Y exp(2Y )
9.101 exp(Y ) z /2
n
n
1
9.103
Yi2
n i=1

9.99 p z /2

(Y )2

i
9.105 2 =
n
9.107 exp(t/Y )
9.109 a N 1 = 2Y 1

N2 1
3n
9.111 252 85.193

Chapter 10
10.3 a c = 11
b .596
c .057
10.5 c = 1.684
10.7 a False
b False
c True
d True
e False
f
i True
10.17
10.21
10.23

10.25
10.27
10.29
10.33
10.35
10.37
10.39
10.41
10.43
10.45
10.47
10.49
10.51
10.53

10.55

ii True
iii False
a H0 : 1 = 2 , Ha : 1 > 2
c z = .075
z = 3.65, reject H0
a-b H0 : 1 2 = 0 vs.
Ha : 1 2
= 0, which
is a twotailed test.
c
z = .954, which does
not lead to a rejection
with = .10.
|z| = 1.105, do not reject
z = .1202, do not reject
z = 4.47
z = 1.50, no
z = 1.48 (1 = homeless), no
approx. 0 (.0000317)
.6700
.025
a .49
b .1056
.22 .155 or (.065, .375)
.5148
129.146, yes
z = 1.58 pvalue = .1142, do not
reject
a z = .996, pvalue = .0618
b No
c z = 1.826, pvalue = .0336
d Yes
z = 1.538; pvalue = .0616; fail to
reject H0 with = .01

10.57 z = 1.732; pvalue = .0836


10.63 a t = 1.341, fail to reject H0
10.65 a t = 3.24, pvalue < .005, yes
b Using the Applet, .00241
c 39.556 3.55
10.67 a t = 4.568 and t.01 = 2.821 so
reject H0 .

b The 99% lower condence bound

54
is 358 2.821 = 309.83.
10
10.69 a t = 1.57, .10 < pvalue <.20,
do not reject; using applet,
pvalue = .13008
i t.10 = 1.319 and
t.05 = 1.714;
.10 < pvalue < .20.
ii Using the Applet,
2P(T < 1.57) =
2(.06504) = .13008.

10.71 a y 1 = 97.856, s12 = .3403,

10.73

10.75
10.77

10.79

y 2 = 98.489, s22 = .3011,


t = 2.3724, t.01 = 2.583,
t.025 = 2.12, so .02 < pvalue
< .05
b Using Applet, .03054
a t = 1.92, do not reject
.05 < pvalue < .10; applet
pvalue = .07084
b t = .365, do not reject pvalue
> .20; applet pvalue = .71936
t = .647, do not reject
a t = 5.54, reject, pvalue < .01;
applet pvalue approx. 0
b Yes
c t = 1.56, .10 < pvalue < .20;
applet pvalue = .12999
d Yes
a 2 = 12.6, do not reject
b .05 < pvalue < .10
c Applet pvalue = .08248

Answers 891

10.83 a 12 =

22
2
b 1 < 22
c 12 > 22
10.85 2 = 22.45, pvalue < .005; applet

10.109 a = 

m+n

b X /Y distributed as F with 2m and

pvalue = .0001

10.89 a .15
b .45
c .75
d 1
10.91 a Reject if Y >= 7.82.
b .2611, .6406, .9131, .9909
10.93 n = 16
4
2
10.95 a U = 20 i=1
Yi has (24)

distribution under H0 : reject H0


if U > 2
b Yes
10.97 d Yes, is UMP
n

10.99 a
Yi k
i=1

b Use Poisson
 table to nd k such
Yi k) =

that P(

c Yes
10.101 a

n


Yi < c

i=1

b Yes

10.103 a Reject H0 if Y(n) 0 n


b Yes
(n

+ (m
has
02
2
(n+m2) distribution under H0 ;
reject if 2 > 2

10.107 2 =

1)S12

1)S22

( X )m (Y )m
m+n
m X + n Y

2n degrees of freedom

10.115 a True
b False
c False
d True
e False
f False
g False
h False
i True
10.117 a t = 22.17, pvalue < .01
b .0105 .001
c Yes
d No
10.119 a H0 : p = .20, Ha : p > .20
b = .0749
10.121 z = 5.24, pvalue approx. 0
10.123 a F = 2.904, no
b (.050, .254)
10.125 a t = 2.657, .02 < pvalue < .05
b 4.542 3.046
10.127 T =


1+a+b
n(3n3)

)(1 2 3 )
( X +Y W
.
1/2


)2
(X i X )2 + a1 (Yi Y )2 + b1 (Wi W

with (3n 3) degrees of freedom


 n
n
i=1 (yi y(1) )
10.129 =

 n n1,0

(yi y(1) )
exp i=1
+n .
1,0

Chapter 11
11.3 y = 1.5 .6x
11.5 y = 21.575 + 4.842x
11.7 a The relationship appears to be
b
c
11.9 b
d
11.11 1
11.13 a
11.17 a
b

proportional to x 2 .
No
No, it is the best linear model.
y = 15.45 + 65.17x
108.373
= 2.514
The least squares line is
y = 452.119 29.402x
SSE = 18.286;
S 2 = 18.286/6 = 3.048
The tted line is
y = 43.35 + 2.42x . The same

11.19 a
c
11.23 a
b
c
d
11.25 a
b
c
d

answer for SSE (and thus S 2 ) is


found.
The least squares line is:
y = 3.00 + 4.75x
s 2 = 5.025
t = 5.20, reject H0
.01 < pvalue < .02
.01382
(.967, .233)
t = 3.791, pvalue < .01
Applet pvalue = .0053
Reject
.475 .289

892

Answers

11.29 T = 1 1

11.33
11.35
11.37
11.39
11.41
11.43
11.45
11.47
11.51
11.53

11.57
11.59
11.61
11.63

11.67
11.69
11.73

1
Scc

11.75 21.9375 3.01


11.77 Following Ex. 11.76, the 95%

PI = 39.9812 213.807
11.79 21.9375 6.17
(SSEY + SSEW )/(n + m 4).
11.83 a F = 21.677, reject
H0 is rejected in favor of Ha for large
b SSE R = 1908.08
values of |T |.
11.85 a F = 40.603, pvalue < .005
t = 73.04, pvalue approx. 0, H0 is
b 950.1676
rejected
11.87 a F = 4.5, F1 = 9.24, fail to
t = 9.62, yes
reject H0
x = x .
c F = 2.353, F1 = 2.23, reject H0
(4.67, 9.63)
11.89 a True
25.395 2.875
b False
b (72.39, 75.77)
c False
(59.73, 70.57)
11.91 F = 10.21
(.86, 15.16)
11.93 90.38 8.42
(.27, .51)
11.95 a y = 13.54 0.053x
t = 9.608, pvalue < .01
b t = 6.86
a r 2 = .682
c .929 .33
b .682
11.97 a y = 1.4825+.5x1 +.1190x2 .5x3
c t = 4.146, reject
b y = 2.0715
d Applet pvalue = .00161
c t = 13.7, reject
a sign for r
d (1.88, 2.26)
b r and n
e (1.73, 2.41)
r = .3783
11.99 If 9 x 9, choose n/2 at x = 9
.979 .104
and n/2 at x = 9.
a 1 = .0095, 0 = 3.603 and
11.101 a y = 9.34 + 2.46x1 + .6x2 + .41x1 x2
1 = (.0095) = .0095,
b 9.34 , 11.80
0 = exp(3.603) = 36.70.
d For bacteria A, y = 9.34. For
Therefore, the prediction equation
bacteria B, y = 11.80. The
is y = 36.70e.0095x .
observed growths were 9.1 and
b The 90% CI for

is
 0
12.2, respectively.
e3.5883 , e3.6171 = (36.17, 37.23)
e 12.81 .37
y = 2.1 .6x
f 12.81 .78
a y = 32.725 + 1.812x
b y = 35.5625 + 1.8119x .1351x 2 11.107 a r = .89
b t = 4.78, pvalue <.01, reject
t = 1.31, do not reject
S

11.31

1
Sx x

 , where S =

Chapter 12
12.1 n 1 = 34, n 2 = 56
12.3 n = 246, n 1 = 93, n 2 = 154
12.5 With n = 6, three rats should receive

x = 2 units and three rats should


receive x = 5 units.
12.11 a This occurs when > 0.
b This occurs when = 0.
c This occurs when < 0.
d Paired better when > 0,
independent better when < 0

12.15 a t = 2.65, reject


12.17 a i
12.31 a i
1

b i , [ P2 + 2 ]
n
c 1 2 , 2 2 /n, normal
12.35 a t = 4.326, .01 < pvalue
< .025

b 1.58 1.014
c 65 pairs

12.37 k1 = k3 = .25; k2 = .50

Answers 893

Chapter 13
13.1 a F = 2.93, do not reject
b .109
c |t| = 1.71, do not reject, F = t 2
13.7 a F = 5.2002, reject
b pvalue = .01068
13.9 SSE = .020; F = 2.0, do not
reject

13.11 SST = .7588; SSE = .7462;

F = 19.83, pvalue < .005, reject

13.49
13.53
13.55
13.57
13.59
13.61
13.63
13.69

13.13 SST = 36.286; SSE = 76.6996;

F = 38.316, pvalue < .005, reject

13.15 F = 63.66, yes, pvalue < .005


13.21 a 12.08 10.96
b Longer
c Fewer degrees of freedom
13.23 a 1.568 .164 or (1.404, 1.732); yes
b (.579, .117); yes
13.25 .28 .102
13.27 a 95% CI for A : 76 8.142

13.29
13.31
13.33
13.35
13.37
13.39
13.41

13.45

or (67.868, 84.142)
b 95% CI for B : 66.33 10.51 or
(55.82, 76.84)
c 95% CI for A B :
9.667 13.295
a 6.24 .318
b .29 .241
a F = 1.32, no
b (.21, 4.21)
(1.39, 1.93)
a 2.7 3.750
b 27.5 2.652
a
b Overall mean
b (2 2 )/b
a F = 3.11, do not reject
b pvalue > .10
c pvalue = .1381
d s D2 = 2MSE
a F = 10.05; reject
b F = 10.88; reject

13.47
Source
Treatments
Blocks
Error
Total

df
SS
MS
F
3 8.1875 2.729
1.40
3 7.1875 2.396
1.23
9 17.5625 1.95139
15 32.9375

F = 1.40, do not reject

13.71
13.73
13.75
13.77
13.79

13.81

F = 6.36; reject
The 95% CI is 2 2.83.
The 95% CI is .145 .179.
The 99% CI is 4.8 5.259.
nA 3
b = 16; n = 48
Sample sizes differ.
a 0 + 3 is the mean response to
treatment A in block III.
b 3 is the difference in mean
responses to chemicals A and D in
block III.
F = 7; H0 is rejected
As homogeneous as possible within
blocks.
b F = 1.05; do not reject
a A 95% CI is .084 .06 or
(.024, .144).
a 16
b 135 degrees of freedom left for
error.
c 14.14
F = 7.33; yes; blocking induces loss in
degrees of freedom for estimating 2 ;
could result in sight loss of information
if block to block variation is small

13.83 a
Source
Treatments
Blocks
Error
Total

df
SS
MS
F
2 524,177.167 262,088.58 258.237
3 173,415
57,805.00
56.95
6
6,089.5
1,014.9167
11 703,681.667

b
c
d
e
f
13.85 a

6
Yes, F = 258.19, pvalue < .005
Yes, F = 56.95, pvalue < .005
22.527
237.25 55.13
SST = 1.212, df = 4
SSE = .571, df = 22
F = 11.68; pvalue < .005
b |t| = 2.73; H0 is rejected; 2(.005)
< pvalue < 2(.01).
13.87 Each interval should have condence
coefcient 1 .05/4 = .9875 .99;
A D : .320 .251
B D : .145 .251
C D : .023 .251
E D : .124 .251

894

Answers

13.89 b
c
13.91 a
b

2
2 = 0
2
; B2 + k1
 k
b
2
2 + k1
i=1 i

c 2 + k B2
d 2

Chapter 14
14.1 a X 2 = 3.696, do not reject
b Applet pvalue = .29622
14.3 X 2 = 24.48, pvalue < .005
14.5 a z = 1.50, do not reject
b Hypothesis suggested by observed
data

14.7 .102 .043


14.9 a .39 .149
b .37 .187, .39 .182, .48 .153
14.11 X 2 = 69.42, reject
14.13 a X 2 = 18.711, reject
b pvalue < .005
c Applet pvalue = .00090
14.15 b X 2 also multiplied by k
14.17 a X 2 = 19.0434 with a pvalue of
.004091.

b X 2 = 60.139 with a pvalue of


approximately 0.

c Some expected counts < 5

X 2 = 22.8705, reject
pvalue < .005
X 2 = 13.99, reject
X 2 = 13.99, reject
X 2 = 1.36, do not reject
X 2 = 19.1723, p-value =
0.003882, reject
c .11 .135
X 2 = 38.43, yes
a X 2 = 14.19, reject
X 2 = 21.51, reject
X 2 = 6.18, reject; .025 < pvalue
< .05
a Yes
b pvalue = .002263
X 2 = 8.56, df = 3; reject
X 2 = 3.26, do not reject
X 2 = 74.85, reject

14.19 a
b
14.21 a
b
c
14.25 b
14.27
14.29
14.31
14.33
14.35
14.37
14.41
14.43

Chapter 15
15.1
Rejection region
M 6 or M 19 P(M 6) +
M 7 or M 18 P(M 7) +
M 8 or M 17 P(M 8) +

P(M 19) = .014


P(M 18) = .044
P(M 17) = .108

15.3 a m = 2, yes
b Variances not equal
15.5 P(M 2 or M 8) = .11, no
15.7 a P(M 2 or M 7) = .18, do
not reject

b t = 1.65, do not reject


15.9 a pvalue = .011, do not reject
15.11 T = min(T + , T ), T = T .
15.13 a T = 6, .02 < pvalue < .05
b T = 6, 0.1 < pvalue < .025
15.15 T = 3.5, .025 < pvalue < .05
15.17 T = 11, reject
15.21 a U = 4; pvalue = .0364
b U = 35; pvalue = .0559
c U = 1; pvalue = .0476

15.23
15.25
15.27
15.29
15.31

15.33
15.37
15.39
15.41

15.45
15.47

U = 9, do not reject
z = 1.80, reject
U = 0, pvalue = .0096
H = 16.974, p-value < .001
a SST = 2586.1333; SSE =
11,702.9; F = 1.33, do not
reject
b H = 1.22, do not reject
H = 2.03, do not reject
a No, pvalue = .6685
b Do not reject H0
Fr = 6.35, reject
a Fr = 65.675, pvalue < .005,
reject
b m = 0, P(M = 0) = 1/256,
pvalue = 1/128
The null distribution is given by
P(Fr = 0) = P(Fr = 4) = 1/6 and
P(Fr = 1) = P(Fr = 3) = 1/3.
R = 6, no

Answers

15.49 a .0256
b An usually small number of runs

15.61 a
b
c
15.63 T

15.51
15.53
15.55

15.65
15.67
15.69
15.71
15.73

15.57
15.59

(judged at = .05) would imply a


clustering of defective items in
time; do not reject.
R = 13, do not reject
r S = .911818; yes.
a r S = .8449887
b Reject
r S = .6768, use two-tailed test, reject
r S = 0; pvalue < .005

Randomized block design


No
pvalue = .04076, yes
= 73.5, do not reject, consistent with
Ex. 15.62
U = 17.5, fail to reject H0
.0159
H = 7.154, reject
Fr = 6.21, do not reject
.10

Chapter 16
16.1 a
b
c
d
e
16.3 c
e
16.7 a
b
16.9 b

16.11 e

(10, 30)
n = 25
(10, 30), n = 25
Yes
Posterior for the (1, 3) prior.
Means get closer to .4, std dev
decreases.
Looks more and more like normal
distribution.
Y +1
n+4
np + 1 np(1 p)
;
n + 4 (n + 4)2
+1
;
+ +Y
( + 1)( + Y 1)
(+ + Y+ 1)(+ + Y 
)
1
n

+
Y
n + 1
n + 1

895

16.13 a (.099, .710)


b Both probabilities are .025
c P(.099 < p < .710) = .95
h Shorter for larger n.
16.15 (.06064, .32665)
16.17 (.38475, .66183)
16.19 (5.95889, 8.01066)
16.21 Posterior probabilities of null and
alternative are .9526 and .0474,
respectively, accept H0 .
16.23 Posterior probabilities of null and
alternative are .1275 and .8725,
respectively, accept Ha .
16.25 Posterior probabilities of null and
alternative are .9700 and .0300,
respectively, accept H0 .

You might also like